a survey of 1457 people, 1107 people said they voted in a recent presidential election. Voting records show that 74% of eligible voters actually did vote. Given that 74% of eligible voters actually did vote, (a) find the probability that among 1457 randomly selected voters, at least 1107 actually did vote. (b) What do the results from part (a) suggest? (a) P(X≥1107)= (Round to four decimal places as needed.)

Answers

Answer 1

(a) P(X ≥ 1107) = 1 - P(X ≤ 1106) = 1 - F(1106),

where X represents the number of voters who voted out of 1457. Using a binomial distribution with n = 1457 and p = 0.74, we can get F(1106) using the formula:

F(x) = P(X ≤ x) = ∑(nCr * p^r * q^(n-r)) for r = 0 to x, where q = 1 - p. Further explanation of (a):

Therefore, we can substitute the values of n, p, and q in the formula, and the values of r from 0 to 1106 to obtain F(1106) as:

F(1106) = P(X ≤ 1106)

= ∑(1457C0 * 0.74^0 * 0.26^1457 + 1457C1 * 0.74^1 * 0.26^1456 + ... + 1457C1106 * 0.74^1106 * 0.26^351)

Now, we can use any software or calculator that can compute binomial cumulative distribution function (cdf) to calculate F(1106). Using a calculator to get the probability, we get:

P(X ≥ 1107) = 1 - P(X ≤ 1106)

= 1 - F(1106) = 1 - 0.999993 ≈ 0.00001 (rounded to four decimal places as needed).

Therefore, the probability that among 1457 randomly selected voters, at least 1107 actually did vote is approximately 0.00001.

(b) The results from part (a) suggest that it is highly unlikely to observe 1107 or more voters who voted out of 1457 randomly selected voters, assuming that the true proportion of voters who voted is 0.74.

This implies that the actual proportion of voters who voted might be less than 0.74 or the sample of 1457 people might not be a representative sample of the population of eligible voters.

To know more about binomial distribution visit:

https://brainly.com/question/29137961

#SPJ11


Related Questions

Find f′ (2), where f(t)=u(t)⋅v(t),u(2)=⟨2,1,−1⟩,u ′(2)=⟨7,0,6⟩, and v(t)=⟨t,t ^2,t^ 3 ⟩. f ′(2)=

Answers

f'(2) = 56. To find f'(2), we need to use the product rule of differentiation. The product rule states that if we have two functions, u(t) and v(t), then the derivative of their product is given by:

(fg)'(t) = f'(t)g(t) + f(t)g'(t),

where f(t) represents u(t) and g(t) represents v(t).

In this case, we have f(t) = u(t) ⋅ v(t), so we can apply the product rule to find f'(t):

f'(t) = u'(t) ⋅ v(t) + u(t) ⋅ v'(t).

Given:

u(2) = ⟨2, 1, -1⟩,

u'(2) = ⟨7, 0, 6⟩,

v(t) = ⟨t, t^2, t^3⟩.

We can substitute these values into the product rule formula:

f'(t) = u'(t) ⋅ v(t) + u(t) ⋅ v'(t).

f'(2) = u'(2) ⋅ v(2) + u(2) ⋅ v'(2).

Let's calculate each part separately:

u'(2) ⋅ v(2) = ⟨7, 0, 6⟩ ⋅ ⟨2, 4, 8⟩ = 7⋅2 + 0⋅4 + 6⋅8 = 14 + 0 + 48 = 62.

u(2) ⋅ v'(2) = ⟨2, 1, -1⟩ ⋅ ⟨1, 2⋅2, 3⋅2^2⟩ = 2⋅1 + 1⋅4 + (-1)⋅12 = 2 + 4 - 12 = -6.

Finally, we can calculate f'(2) by adding the two results:

f'(2) = u'(2) ⋅ v(2) + u(2) ⋅ v'(2) = 62 + (-6) = 56.

Therefore, f'(2) = 56.

To know more about differentiation visit :

https://brainly.com/question/31539041

#SPJ11

A friend offers you a free ticket to a concert, which you decide to attend. The concert takes 4 hours and costs you $15 for transportation. If you had not attended the concert, you would have worked at your part-time job earning $15 per hour. What is the true cost of you attending the concert?

Answers

The true cost of you attending the concert is $60.

The correct answer for the given problem is as follows:

Opportunity cost is the true cost of you attending the concert.

The reason being, the person had to give up an alternative use of their time to attend the concert.

In the given situation, if the person had not attended the concert they would have worked at their part-time job earning $15 per hour.

Thus, the opportunity cost for attending the concert is equal to the amount of money you would have earned had you not gone to the concert.

So, the opportunity cost of attending the concert would be: $15/hour × 4 hours = $60

The true cost of you attending the concert is $60.

Know more about cost:

https://brainly.com/question/17120857

#SPJ11

"
Gym A charges $18 per month plus a $25 fee. Gym B charges $6 per month plus a $97 fee. a. Gym A and B will cost the same at _________________________ months. b. How much will it cost at that time?
"

Answers

a. Gym A and B will cost the same at 11 months.

b. It will cost $223.00 at that time.

Let's calculate the cost of each gym and find out the time at which both gyms will cost the same.

Gym A cost = $18 per month + $25 fee

Gym B cost = $6 per month + $97 fee

Let's find out when the costs of Gym A and Gym B will be the same.18x + 25 = 6x + 97   (where x represents the number of months)18x - 6x = 97 - 2512x = 72x = 6Therefore, Gym A and Gym B will cost the same after 6 months.

Let's put x = 11 months to calculate the cost of both gyms at that time.

Cost of Gym A = 18(11) + 25 = $223.00Cost of Gym B = 6(11) + 97 = $223.00

Therefore, it will cost $223.00 for both gyms at 11 months.

Learn more about cost:https://brainly.com/question/28147009

#SPJ11

teachers get courses assigned to teach each semester. for each instructor, there are the courses that the instructor can teach based on the skill set of the instructor, and there are courses that the teacher would rather teach all the time, closer to their specialization. 282 probability for data scientists to be able to teach in any department, a teacher must be able to teach more than the favorite courses. let x denote the proportion of teachers who teach the whole spectrum of courses taught in a department, and y the proportion of teachers who teach the courses they specialize in. let x and y have the joint density function f (x,y)=2(x+y), 0

Answers

The probability that a teacher can teach in any department is 2/3.

How to find the probability

To find the probability that a teacher can teach in any department,

find the proportion of teachers who teach the whole spectrum of courses taught in a department, which is denoted by x.

Let's denote the proportion of teachers who can teach their favorite courses by y.

The joint density function of x and y is given by

[tex]f(x,y) = 2(x+y), 0 < x < 1, 0 < y < 1, and x + y < 1[/tex]

To find the probability that a teacher can teach in any department, integrate the joint density function over the region where x > y:

[tex]P(x > y) = \int\int(x > y) f(x,y) dxdy[/tex]

Split the integration into two parts: one over the region where y varies from 0 to x, and another over the region where y varies from x to 1:

[tex]P(x > y) = \int[0,1]\int[0,x] 2(x+y) dydx + \int[0,1]\int[x,1-x] 2(x+y) dydx\\P(x > y) = \int[0,1] x^2 + 2x(1-x) dx\\= \int[0,1] (2x - x^2) dx\\= [x^2 - x^3/3]_0^1[/tex]

= 2/3

Therefore, the probability that a teacher can teach in any department is 2/3.

Learn more on joint density function on https://brainly.com/question/31266281

#SPJ4

How can thee model be ued to determine 1. 42−0. 53? Enter your anwer in the boxe. You cannot ubtract 5 tenth from 4 tenth or 3 hundredth from 2 hundredth, o regroup one whole into 10 tenth and then regroup one tenth into 10 hundredth. There are now 0 whole, tenth, and hundredth. After removing 5 tenth and 3 hundredth, there are tenth and hundredth remaining. Therefore, the difference of 1. 42 and 0. 53 i

Answers

The difference between 1.42 and 0.53 is 0.37.

The model can be used to determine the difference between 1.42 and 0.53.

First, we start with 1 whole and 4 tenths (1.4) and represent it in the model. Next, we subtract 5 tenths (0.5) from 4 tenths (0.4). Since we cannot subtract directly, we need to regroup. We can regroup 1 whole into 10 tenths and then regroup 1 tenth into 10 hundredths. Now we have 10 tenths (1) and 40 hundredths (0.4).

Next, we subtract 3 hundredths (0.03) from 40 hundredths (0.4). This can be done directly since the place values match. Subtracting, we get 37 hundredths (0.37).

Therefore, the difference between 1.42 and 0.53 is 0.37.

To summarize, we regrouped to subtract 5 tenths from 4 tenths, and then subtracted 3 hundredths from 40 hundredths. The final answer is 0.37.

Learn more about subtraction using models :

https://brainly.com/question/32595757

#SPJ11

Rewrite each of the following linear differential equations in standard form y'+p(t)y = g(t). Indicate p(t).
(a) 3y'-2t sin(t) = (1/t)y
(b) y'-t-ty=0
(c) e^t y' = 5+ y

Answers

(A) [tex]\(S'(t) = 0.12t^2 + 0.8t + 2\).  \(S(2) = 12.88\)[/tex]

(B) [tex]\(S'(2) = 4.08\)[/tex] (both rounded to two decimal places).

(C) The interpretation of \(S'(10) = 22.00\) is that after 10 months, the rate of change of the total sales with respect to time is 22 million dollars per month

(A) To find \(S'(t)\), we need to take the derivative of the function \(S(t)\) with respect to \(t\).

[tex]\(S(t) = 0.04t^3 + 0.4t^2 + 2t + 5\)[/tex]

Taking the derivative term by term, we have:

[tex]\(S'(t) = \frac{d}{dt}(0.04t^3) + \frac{d}{dt}(0.4t^2) + \frac{d}{dt}(2t) + \frac{d}{dt}(5)\)[/tex]

Simplifying each term, we get:

\(S'(t) = 0.12t^2 + 0.8t + 2\)

Therefore, \(S'(t) = 0.12t^2 + 0.8t + 2\).

(B) To find \(S(2)\), we substitute \(t = 2\) into the expression for \(S(t)\):

[tex]\(S(2) = 0.04(2)^3 + 0.4(2)^2 + 2(2) + 5\)\(S(2) = 1.28 + 1.6 + 4 + 5\)\(S(2) = 12.88\)[/tex]

To find \(S'(2)\), we substitute \(t = 2\) into the expression for \(S'(t)\):

[tex]\(S'(2) = 0.12(2)^2 + 0.8(2) + 2\)\(S'(2) = 0.48 + 1.6 + 2\)\(S'(2) = 4.08\)[/tex]

Therefore, \(S(2) = 12.88\) and \(S'(2) = 4.08\) (both rounded to two decimal places).

(C) The interpretation of \(S(10) = 105.00\) is that after 10 months, the total sales of the company are expected to be $105 million. This represents the value of the function [tex]\(S(t)\) at \(t = 10\)[/tex].

The interpretation of \(S'(10) = 22.00\) is that after 10 months, the rate of change of the total sales with respect to time is 22 million dollars per month. This represents the value of the derivative \(S'(t)\) at \(t = 10\). It indicates how fast the sales are increasing at that specific time point.

Learn more about interpretation here:-

https://brainly.com/question/27749887

#SPJ11

PV81-x²
where x represents the number of hundreds of canisters and p is the price, in dollars, of a single canister.
(a) If p = 7, find the corresponding value of x.
x=11

Answers

The corresponding value of x when p = 7 is x = 11.

Given the equation PV = 81 - x², where x represents the number of hundreds of canisters and p is the price of a single canister in dollars.

To find the corresponding value of x when p = 7, we substitute p = 7 into the equation:

7V = 81 - x²

Rearranging the equation:

x² = 81 - 7V

To find the corresponding value of x, we need to know the value of V. Without the specific value of V, we cannot determine the exact value of x.

However, if we are given additional information about V, we can substitute it into the equation and solve for x. In this case, if the value of V is such that 7V is equal to 81, then the equation becomes:

7V = 81 - x²

Since 7V is equal to 81, we have:

7(1) = 81 - x²

7 = 81 - x²

Rearranging the equation:

x² = 81 - 7

x² = 74

Taking the square root of both sides:

x = ±√74

Since x represents the number of hundreds of canisters, the value of x must be positive. Therefore, the corresponding value of x when p = 7 is x = √74, which is approximately equal to 8.60. However, it's important to note that without additional information about the value of V, we cannot determine the exact value of x.

Learn more about corresponding value here:

brainly.com/question/12682395

#SPJ11

The five number summary of a data set was found to be: \[ 46,54,60,65,70 \] What is the interquartile range?

Answers

The interquartile range for the given data set is 17.5.

Given, The five number summary of a data set was found to be: \[ 46,54,60,65,70 \].

The interquartile range (IQR) can be calculated using the following formula:

IQR = Q3 - Q1,

where Q3 represents the third quartile, and Q1 represents the first quartile.

To find the interquartile range (IQR), let us first find the first quartile and the third quartile of the data set:

First Quartile (Q1):

Median of the lower half of the data set \[ 46, 54 \]

Median = (46 + 54) / 2 = 50

Third Quartile (Q3):

Median of the upper half of the data set \[ 65, 70 \]

Median = (65 + 70) / 2 = 67.5

Using the values obtained, we can now calculate the interquartile range (IQR) as follows:

IQR = Q3 - Q1

IQR = 67.5 - 50

IQR = 17.5

Therefore, the interquartile range for the given data set is 17.5.

Learn more about interquartile range: https://brainly.com/question/1210750

#SPJ11

Rachel gets a starting salavy of P^(6000) amonth, and an increase of perd annually. What will be her salary durieg the fifth year?

Answers

Rachel's salary during the fifth year will be P^(8316.15) per month. Rachel gets a starting salary of P^(6000) per month and an increase of p% annually.

We are required to calculate her salary during the fifth year. To calculate the salary during the fifth year, we need to find out the salary for each of the five years. The salary during the first year will be P^(6000), and the salary during the second year can be calculated as follows:

Salary after the first year = P^(6000) + P^(6000) × p/100

= P^(6000) × (1 + p/100)

Similarly, the salary during the third year will be: Salary after the second year = P^(6000) × (1 + p/100) + P^(6000) × (1 + p/100) × p/100

= P^(6000) × (1 + p/100)^2

Similarly, we can calculate the salaries for the fourth and fifth years as: Salary after the third year = P^(6000) × (1 + p/100)^3

Salary after the fourth year = P^(6000) × (1 + p/100)^4

Salary after the fifth year = P^(6000) × (1 + p/100)^5

Given that Rachel gets an increase of p% annually, we can use the compound interest formula to calculate the value of p as follows:

We know that P^(8316.15) = P^(6000) × (1 + p/100)^5

Taking the fifth root on both sides, we get:1 + p/100 = (P^(8316.15) / P^(6000))^(1/5)

Substituting the values, we get:1 + p/100 = (1.3817217)

The value of p can be calculated as follows: p/100 = 0.3817217p = 38.17217%

Thus, Rachel's salary during the fifth year will be P^(8316.15) per month, which is approximately P^(8316).

To know more about salary visit:

https://brainly.com/question/29105365

#SPJ11

A fi making toaster ovens finds that the total cost, C(x), of producing x units is given by C(x) = 50x + 310. The revenue, R(x), from selling x units is deteined by the price per unit times the number of units sold, thus R(x) = 60x. Find and interpret (R - C)(64).

Answers

The company makes a profit of $570 by producing and selling 64 units.Given that the cost of producing x units is given by C(x) = 50x + 310 and revenue from selling x units is determined by the price per unit times the number of units sold, thus R(x) = 60x.

To find and interpret (R - C)(64).

Solution:(R - C)(64) = R(64) - C(64)R(x) = 60x, therefore R(64) = 60(64) = $3840.C(x) = 50x + 310, therefore C(64) = 50(64) + 310 = $3270

Hence, (R - C)(64) = R(64) - C(64) = 3840 - 3270 = $570.

Therefore, the company makes a profit of $570 by producing and selling 64 units.

For more question on revenue

https://brainly.com/question/23706629

#SPJ8

There are 12 points A,B,… in a given plane, no three on the same line. The number of triangles are determined by the points such that contain the point A as a vertex is: (a) 65 (b) 55 (c) 75 (d) 66

Answers

The answer is (c) 75. The number of triangles that can be formed using the points A, B, and C as vertices is 1. We can then choose the remaining vertex from the 9 points that are not A, B, or C. This gives us a total of 9 possible choices for D.

Therefore, the number of triangles that contain A as a vertex is 1 * 9 = 9.

Similarly, we can count the number of triangles that contain B, C, D, E, F, G, H, I, J, K, and L as vertices by considering each point in turn as one of the vertices. For example, to count the number of triangles that contain B as a vertex, we can choose two other points from the 10 remaining points (since we cannot use A or B again), which gives us a total of (10 choose 2) = 45 possible triangles. We can do this for each of the remaining points to get:

Triangles containing A: 9

Triangles containing B: 45

Triangles containing C: 45

Triangles containing D: 36

Triangles containing E: 28

Triangles containing F: 21

Triangles containing G: 15

Triangles containing H: 10

Triangles containing I: 6

Triangles containing J: 3

Triangles containing K: 1

Triangles containing L: 0

The total number of triangles is the sum of these values, which is:

9 + 45 + 45 + 36 + 28 + 21 + 15 + 10 + 6 + 3 + 1 + 0 = 229

However, we have counted each triangle three times (once for each of its vertices). Therefore, the actual number of triangles is 229/3 = 76.33, which is closest to option (c) 75.

Therefore, the answer is (c) 75.

learn more about triangles here

https://brainly.com/question/2773823

#SPJ11

Which of the following gives the equation of a circle of radius 22 and center at the point (-1,2)(-1,2)?

Answers

Step-by-step explanation:

Equation of a circle is

[tex](x - h) {}^{2} + (y - k) {}^{2} = {r}^{2} [/tex]

where (h,k) is the center

and the radius is r.

Here the center is (-1,2) and the radius is 22

[tex](x + 1) {}^{2} + (y - 2) {}^{2} = 484[/tex]

There are functions of the form x^{r} that solve the differential equation x²y"-6xy' + 10 y=0
Give the solution to the initial value problem [x²y"-6xy' + 10 y=0 y(1)=0 y'(1)=3]

Answers

The solution in mathematical notation:

y = x² - 1

The differential equation x²y"-6xy' + 10 y=0 is an Euler equation, which means that it can be written in the form αx² y′′ + βxy′ + γ y = 0. The general solution of an Euler equation is of the form y = x^r, where r is a constant to be determined.

In this case, we can write the differential equation as x²(r(r - 1))y + 6xr y + 10y = 0. If we set y = x^r, then this equation becomes x²(r(r - 1) + 6r + 10) = 0. This equation factors as (r + 2)(r - 5) = 0, so the possible values of r are 2 and -5.

The function y = x² satisfies the differential equation, so one solution to the initial value problem is y = x². The other solution is y = x^-5, but this solution is not defined at x = 1. Therefore, the only solution to the initial value problem is y = x².

To find the solution, we can use the initial conditions y(1) = 0 and y'(1) = 3. We have that y(1) = 1² = 1 and y'(1) = 2² = 4. Therefore, the solution to the initial value problem is y = x² - 1.

Here is the solution in mathematical notation:

y = x² - 1

This solution can be verified by substituting it into the differential equation and checking that it satisfies the equation.

Learn more about mathematical notation here:-

https://brainly.com/question/31065745

#SPJ11

Determine the coefficient of each term, 9x^(7)+x^(5)-3x^(3)+6 The coefficient of the term 9x^(7) is

Answers

The coefficient of the term 9x^7 is 9. In the given polynomial expression, the term 9x^7 represents the product of the coefficient (9) and the variable raised to the power of 7 (x^7).

In the polynomial expression 9x^7 + x^5 - 3x^3 + 6, each term consists of a coefficient and a variable raised to a certain power. The coefficient represents the numerical factor multiplied by the variable term. In the term 9x^7, the coefficient is 9. This means that the variable x is multiplied by 9 raised to the power of 7, resulting in 9x^7.

The coefficient of a term determines the scale or magnitude of that term within the polynomial expression. It indicates the amount by which the term contributes to the overall value of the expression. In this case, the coefficient of 9 in 9x^7 implies that the term 9x^7 has a greater impact on the polynomial's value compared to other terms, such as x^5, -3x^3, and 6.

To know more about coefficient refer here:

https://brainly.com/question/13431100

#SPJ11

Find the computational complexity for the following four loo a. for (cnt1=0,i=1;i⇔=n i

i++) for (j=1;j<=n;j++) cnt1++; b. for (cnt2=0,i=1;i<=n i

i++) for (j=1;j<=i;j++) cnt2++; c. for (cnt3=0,i=1;i⇔n;i∗=2) for (j=1;j<=n;j++) cnt3++; d. for (cnt4 =0,i=1;i⇔=n;i∗=2 ) for (j=1;j<=i;j++) cnt4++;

Answers

The computational complexity for the given loops are as follows:

a. O(n^2)

b. O(n^2)

c. O(n log(n))

d. O(n).

Computational complexity for the following four loops are:

a. Loop 1: for (cnt1=0,i=1;i<=n;i++) for (j=1;j<=n;j++) cnt1++;

Here, there are 2 loops with complexity O(n) and O(n), so the total computational complexity is O(n^2).

b. Loop 2: for (cnt2=0,i=1;i<=n;i++) for (j=1;j<=i;j++) cnt2++;

Here, the first loop has complexity O(n) and the second loop is O(i) where i varies from 1 to n.

Hence, the total computational complexity of this loop is O(n^2).

c. Loop 3: for (cnt3=0,i=1;i<=n;i*=2) for (j=1;j<=n;j++) cnt3++;

Here, the first loop is O(log(n)) because i is multiplied by 2 in each iteration until i becomes greater than n.

The second loop is O(n), so the total computational complexity is O(n log(n)).

d. Loop 4: for (cnt4 =0,i=1;i<=n;i*=2) for (j=1;j<=i;j++) cnt4++;

Here, the first loop is O(log(n)) and the second loop is O(i) where i varies from 1 to n.

Hence, the total computational complexity of this loop is O(n).

Thus, the computational complexity for the given loops are as follows:

a. O(n^2)

b. O(n^2)

c. O(n log(n))

d. O(n).

Note: The computational complexity of an algorithm is the amount of resources it requires to run. It is usually expressed in terms of the input size.

To know more about iteration, visit:

https://brainly.com/question/31197563

#SPJ11

Find The General Solution To Y′′+12y′+36y=0.

Answers

Given y′′+12y′+36y=0 We can solve the above second order differential equation by finding the characteristic equation as: r^2 + 12r + 36 = 0

Now, let us find the roots of the above equation: \begin{aligned} r^2 + 6r + 6r + 36 &= 0 \\

\Rightarrow r(r+6) + 6(r+6) &= 0 \\

\Rightarrow (r+6)(r+6) &= 0 \\

\Rightarrow (r+6)^2 &= 0 \end{aligned}

So, we got the repeated roots as r = -6. As the roots are repeated we can write the general solution of the given differential equation as: y(x) = (c_1 + c_2 x) e^{-6x}  

Here c1 and c2 are constants. Hence the general solution of the given second order differential equation is

y(x) = (c1 + c2 x) e^{-6x}.

The given differential equation is y′′+12y′+36y=0.

So, the general solution of the given differential equation is y(x) = (c1 + c2 x) e^{-6x} with c1, c2 being constants.

To know more about equation visit:

https://brainly.com/question/29657983

#SPJ11

Find, correct to the nearest degree, the three angles of the triangle with the given vertices. A(1,0,−1),B(5,−3,0),C(1,2,5) ∠CAB= ∠ABC= ∠BCA=

Answers

The angles of the triangle with the given vertices are approximately: ∠CAB ≈ 90 degrees ∠ABC ≈ 153 degrees ∠BCA ≈ 44 degrees.

To find the angles of the triangle with the given vertices, we can use the dot product and the arccosine function.

Let's first find the vectors AB, AC, and BC:

AB = B - A

= (5, -3, 0) - (1, 0, -1)

= (4, -3, 1)

AC = C - A

= (1, 2, 5) - (1, 0, -1)

= (0, 2, 6)

BC = C - B

= (1, 2, 5) - (5, -3, 0)

= (-4, 5, 5)

Next, let's find the lengths of the vectors AB, AC, and BC:

|AB| = √[tex](4^2 + (-3)^2 + 1^2)[/tex]

= √26

|AC| = √[tex](0^2 + 2^2 + 6^2)[/tex]

= √40

|BC| = √[tex]((-4)^2 + 5^2 + 5^2)[/tex]

= √66

Now, let's find the dot products of the vectors:

AB · AC = (4, -3, 1) · (0, 2, 6)

= 4(0) + (-3)(2) + 1(6)

= 0 - 6 + 6

= 0

AB · BC = (4, -3, 1) · (-4, 5, 5)

= 4(-4) + (-3)(5) + 1(5)

= -16 - 15 + 5

= -26

AC · BC = (0, 2, 6) · (-4, 5, 5)

= 0(-4) + 2(5) + 6(5)

= 0 + 10 + 30

= 40

Now, let's find the angles:

∠CAB = cos⁻¹(AB · AC / (|AB| |AC|))

= cos⁻¹(0 / (√26 √40))

≈ 90 degrees

∠ABC = cos⁻¹(AB · BC / (|AB| |BC|))

= cos⁻¹(-26 / (√26 √66))

≈ 153 degrees

∠BCA = cos⁻¹(AC · BC / (|AC| |BC|))

= cos⁻¹(40 / (√40 √66))

≈ 44 degrees

To know more about triangle,

https://brainly.com/question/33150747

#SPJ11

Post Test: Solving Quadratic Equations he tlles to the correct boxes to complete the pairs. Not all tlles will be used. each quadratic equation with its solution set. 2x^(2)-8x+5=0,2x^(2)-10x-3=0,2

Answers

The pairs of quadratic equations with their respective solution sets are:(1) `2x² - 8x + 5 = 0` with solution set `x = {2 ± (sqrt(6))/2}`(2) `2x² - 10x - 3 = 0` with solution set `x = {5 ± sqrt(31)}/2`.

The solution of each quadratic equation with its corresponding equation is given below:Quadratic equation 1: `2x² - 8x + 5 = 0`The quadratic formula for the equation is `x = [-b ± sqrt(b² - 4ac)]/(2a)`Comparing the equation with the standard quadratic form `ax² + bx + c = 0`, we can say that the values of `a`, `b`, and `c` for this equation are `2`, `-8`, and `5`, respectively.Substituting the values in the quadratic formula, we get: `x = [8 ± sqrt((-8)² - 4(2)(5))]/(2*2)`Simplifying the expression, we get: `x = [8 ± sqrt(64 - 40)]/4`So, `x = [8 ± sqrt(24)]/4`Now, simplifying the expression further, we get: `x = [8 ± 2sqrt(6)]/4`Dividing both numerator and denominator by 2, we get: `x = [4 ± sqrt(6)]/2`Simplifying the expression, we get: `x = 2 ± (sqrt(6))/2`Therefore, the solution set for the given quadratic equation is `x = {2 ± (sqrt(6))/2}`Quadratic equation 2: `2x² - 10x - 3 = 0`Comparing the equation with the standard quadratic form `ax² + bx + c = 0`, we can say that the values of `a`, `b`, and `c` for this equation are `2`, `-10`, and `-3`, respectively.We can use either the quadratic formula or factorization method to solve this equation.Using the quadratic formula, we get: `x = [10 ± sqrt((-10)² - 4(2)(-3))]/(2*2)`Simplifying the expression, we get: `x = [10 ± sqrt(124)]/4`Now, simplifying the expression further, we get: `x = [5 ± sqrt(31)]/2`Therefore, the solution set for the given quadratic equation is `x = {5 ± sqrt(31)}/2`Thus, the pairs of quadratic equations with their respective solution sets are:(1) `2x² - 8x + 5 = 0` with solution set `x = {2 ± (sqrt(6))/2}`(2) `2x² - 10x - 3 = 0` with solution set `x = {5 ± sqrt(31)}/2`.

Learn more about equation :

https://brainly.com/question/29657992

#SPJ11

If the area of a circle is 821 what is the radius

Answers

Answer: r≈16.17

Step-by-step explanation: r=A

π=821

π≈16.16578

Polly bought a package of 5 oatmeal cookies. The total weight of the cookies was 0.9 ounces. How much did each cookie weigh?

Answers

If Polly bought a package of 5 oatmeal cookies and the total weight of the cookies was 0.9 ounces, then each cookie weighs 0.18 ounces.

To calculate the weight of each cookie, follow these steps:

The formula to calculate the weight of each cookie is as follows: Weight of each cookie = Total weight of the cookies / Number of cookies in the package.Substituting total weight= 0.9 ounces and the number of cookies= 5 in the formula, we get the weight of each cookie = 0.9 / 5 ⇒Weight of each cookie = 0.18 ounces.

Therefore, each oatmeal cookie weighs 0.18 ounces.

Learn more about weight:

brainly.com/question/86444

#SPJ11

A population of squirrels grows exponentially at a rate of 4.2 percent per year. The population was 8400 in 2002. Step 1 of 3: Find the exponential function that represents the population t years after 2002. Answer Point f(t) =

Answers

Answer:

P(t) = 8,400e^(0.042)t

P(t) = total population t years after 2002

8,400 initial population at 2002

0.042 = rate of growth

t = #years after 2002

Step-by-step explanation:

Using the formula for exponential growth, in this case P(t) = P(subscript 0) e^(kt), k as rate.

P(subscript 0) initial population = 8400

k rate = 4.2% = 0.042

Plug in the numbers as given by the problem.

Suppose that we have a bulbs box containing 60 bulbs, of which 13 are defective. 2 bulbs are slected at random, with replacement from the box (Round your answer to three decimals) A) Find the probability that both bulbs are defective. B) Find the probability that atleast one of them is defective.

Answers

a) The probability that both bulbs are defective is approximately 0.047.

b) The probability that at least one of the bulbs is defective is approximately 0.386. These probabilities were calculated using the binomial distribution with n = 2 and p = 13/60 for defective bulbs.

We can use the binomial distribution to solve this problem. Let X be the number of defective bulbs in a sample of size 2, with replacement. Then X follows a binomial distribution with n = 2 and p = 13/60 for defective bulbs.

a) The probability that both bulbs are defective is:

P(X = 2) = (2 choose 2) * (13/60)^2 * (47/60)^0

= 1 * (169/3600) * 1

= 169/3600

≈ 0.047

Therefore, the probability that both bulbs are defective is approximately 0.047.

b) The probability that at least one of the bulbs is defective is:

P(X ≥ 1) = 1 - P(X = 0)

= 1 - (2 choose 0) * (13/60)^0 * (47/60)^2

= 1 - 1 * 1 * (2209/3600)

= 1391/3600

≈ 0.386

Therefore, the probability that at least one of the bulbs is defective is approximately 0.386.

learn more about binomial distribution here

https://brainly.com/question/29137961

#SPJ11

the quotient of 3 and a number m foula r=(d)/(t), where d is the distance in miles, r is the rate, and t is the time in hours, at whic tyou travel to cover 337.5 miles in 4.5 hours? (0pts )55mph (0 pts ) 65mph (1 pt) 75mph X (0 pts ) 85mph

Answers

If the formula r= d/t where d is the distance in miles, r is the rate, and t is the time in hours, you can travel at a rate of 75mph to cover 337.5 miles in 4.5 hours.

To calculate at which rate you travel to cover 337.5 miles in 4.5 hours, follow these steps:

The formula r= d/t, where d is the distance in miles, r is the rate, and t is the time in hours.Substituting the values in the formula, we get r= 337.5/ 4.5= = 75mph.

Therefore, at a rate of 75 miles per hour, you can travel to cover 337.5 miles in 4.5 hours.

Learn more about rate:

brainly.com/question/119866

#SPJ11

How to plot the function 2x+1 and 3x ∧
2+2 for x=−10:1:10 on the same plot. x=−10:1:10;y1=2 ∗
x+1;y2=3 ∗
x. ∧
2+2;plot(x,y1,x,y2) x=−10:1:10;y1=2 ∗
x+1;y2=3 ∗
x,a ∧
2+2; plot( x,y1); hold on: plot( x,y2) x=−10:1:10;y1=2 ∗
x+1;y2=3 ∗
x. ∧
2+2;plot(x,y1); plot (x,y2) Both a and b What is the syntax for giving the tag to the x-axis of the plot xlabel('string') xlabel(string) titlex('string') labelx('string') What is the syntax for giving the heading to the plot title('string') titleplot(string) header('string') headerplot('string') For x=[ 1

2

3

] and y=[ 4

5

6], Divide the current figure in 2 rows and 3 columns and plot vector x versus vector y on the 2 row and 2 column position. Which of the below command will perform it. x=[123];y=[45 6]; subplot(2,3,1), plot(x,y) x=[123]:y=[45 6): subplot(2,3,4), plot (x,y) x=[123]:y=[456]; subplot(2,3,5), plot(x,y) x=[123];y=[456]; subplot(3,2,4), plot( (x,y) What is the syntax for giving the tag to the y-axis of the plot ylabel('string') ylabel(string) titley('string') labely('string')

Answers

To plot the function 2x+1 and 3x^2+2 for x = -10:1:10 on the same plot, we will use the following command:

x = -10:1:10;

y1 = 2*x + 1;

y2 = 3*x.^2 + 2;

plot(x, y1);

plot(x, y2)

This will plot both functions on the same graph.

To tag the x-axis of the plot, we can use the command `xlabel('string')`, and to tag the y-axis, we can use `ylabel('string')`.

Therefore, the syntax for giving the tag to the x-axis is `xlabel('string')`, and the syntax for giving the tag to the y-axis is `ylabel('string')`.

We can provide a heading to the plot using the command `title('string')`. Hence, the syntax for giving the heading to the plot is `title('string')`.

To plot vector x versus vector y in the 2nd row and 2nd column position, we use the command `subplot(2, 3, 4), plot(x, y)`. Therefore, the correct option is:

x = [123];

y = [456];

subplot(3, 2, 4);

plot(x, y).

To know more about command visit:

https://brainly.com/question/32329589

#SPJ11

Let X represent the full height of a certain species of tree. Assume that X has a normal probability distribution with μ=245.3 ft and σ=38.9 ft.

You intend to measure a random sample of n=238 trees.

What is the mean of the distribution of sample means?


What is the standard deviation of the distribution of sample means (i.e., the standard error in estimating the mean)?
(Report answer accurate to 2 decimal places.)

Answers

Mean of the distribution of sample means = 245.3 Standard deviation of the distribution of sample means (i.e., the standard error in estimating the mean) = 2.52

The given normal probability distribution is: X =  N(μ = 245.3, σ = 38.9)The sample size is: n = 238. We need to find out the mean and the standard deviation of the distribution of sample means. The formula for the mean of the distribution of sample means is: µx = µ = 245.3Therefore, the mean of the distribution of sample means is 245.3. The formula for the standard deviation of the distribution of sample means is: σx = σ / √n = 38.9 / √238 = 2.52 (rounded to 2 decimal places) Therefore, the standard deviation of the distribution of sample means (i.e., the standard error in estimating the mean) is 2.52 (rounded to 2 decimal places).

Learn more about Standard deviation

https://brainly.com/question/29115611

#SPJ11

For each of the following recurrences, sketch its recursion tree and guess a good asymptotic upper bound on its solution. Then use the substitution method to verify your answer.
a. T(n) = T(n/2) + n3
b. T(n) = 4T(n/3) + n
c. T(n) = 4T(n/2) + n
d. T(n) = 3T (n -1) + 1

Answers

The asymptotic upper bounds for the given recurrence relations are: (a) O(n^3 * log(n)), (b) O(n^log_3(4)), (c) O(n^2 * log(n)), and (d) O(n). The substitution method can be used to verify these bounds.

(a) For the recurrence relation T(n) = T(n/2) + n^3, the recursion tree will have log(n) levels with n^3 work done at each level. Therefore, the total work done can be approximated as O(n^3 * log(n)). This can be verified using the substitution method.

(b) In the recurrence relation T(n) = 4T(n/3) + n, the recursion tree will have log_3(n) levels with n work done at each level. Therefore, the total work done can be approximated as O(n^log_3(4)) using the Master Theorem. This can also be verified using the substitution method.

(c) The recurrence relation T(n) = 4T(n/2) + n will have a recursion tree with log_2(n) levels and n work done at each level. Hence, the total work done can be approximated as O(n^2 * log(n)) using the Master Theorem. This can be verified using the substitution method.

(d) The recurrence relation T(n) = 3T(n-1) + 1 will result in a recursion tree with n levels and constant work done at each level. Therefore, the total work done can be approximated as O(n). This can be verified using the substitution method.

Visit here to learn more about substitution method:

brainly.com/question/22340165

#SPJ11

What is the missing reason in the proof?
Given: AABE ACDE
Prove: ABCD is a parallelogram.
Statement
1. AABE is congruent to ACDE.
2. BE is congruent to DE
AE is congruent to CE.
3. AC and BD bisect each other.
4. ABCD is a parallelogram.
A. Opposite sides property
B. CPCTC
Reason
Given
CPCTC
?
Converse of diagonals theorem

Answers

The missing reason in the proof is determined as the:

converse of the diagonals theorem

What is the Converse of diagonals theorem?

The converse of the diagonals theorem states that if the diagonals of a quadrilateral bisect each other, then the quadrilateral is a parallelogram.

The proof is shown in the image attached below, which shows that in step 3, the diagonals of the quadrilateral bisect each other. Therefore, based on the converse of the diagonals theorem, we can conclude that the quadrilateral is a parallelogram.

Learn more about  Converse of diagonals theorem on:

https://brainly.com/question/17142779

#SPJ1

Find The Area Bounded By The First Quadrant Loop Of The Curve X^5+Y^5=3xy

Answers

The area bounded by the first quadrant loop of the curve x^5 + y^5 = 3xy is approximately 0.536 square units.

To find the area bounded by the curve x^5 + y^5 = 3xy in the first quadrant, we can use the double integral. However, this particular curve is quite complicated to work with directly. Instead, we can use a change of variables to simplify the equation.

Let's make the substitution u = x^5 and v = y^5. Then, we can express the curve equation in terms of u and v:

u + v = 3uv

This is a much simpler equation to work with. Now, let's find the limits of integration for u and v. Since we are considering the first quadrant, both u and v must be positive. From the original equation, we can see that when x = 0, y = 0, and when y = 0, x = 0. Therefore, the limits of integration for u and v are both from 0 to 1.

Now, we can calculate the area using the double integral:

A = ∬R dA

A = ∫∫R du dv

A = ∫[0,1] ∫[0,1] du dv

A = ∫[0,1] u=0 to 1 v=0 to 1 du dv

A = ∫[0,1] (v/2 + v^2/3) u=0 to 1 dv

A = ∫[0,1] (1/2 + v/3) dv

A = (1/2)v + (1/6)v^2 from 0 to 1

A = (1/2)(1) + (1/6)(1^2) - (1/2)(0) - (1/6)(0^2)

A = 1/2 + 1/6

A = 3/6 + 1/6

A = 4/6

A ≈ 0.667 square units

Therefore, the area bounded by the first quadrant loop of the curve x^5 + y^5 = 3xy is approximately 0.667 square units.

Learn more about curve equation here:

brainly.com/question/31467851

#SPJ11

Let X⊆R^d be a set of d+1 affinely independent points. Show that int(conv(X))=∅.
a) √(1/3)a³. √12a² : √2a b) √(27x³y^{5}) : √(1/3)xy
d) 3x.(√27x^{5} : √(1/3)x³)

Answers

We have proved that if X ⊆ R^d is a set of d+1 affinely independent points, then int(conv(X)) ≠ ∅.

Given that X ⊆ R^d is a set of d+1 affinely independent points, we need to prove that int(conv(X)) ≠ ∅.

Definition: A set of points in Euclidean space is said to be affinely independent if no point in the set can be represented as an affine combination of the remaining points in the set.

Solution:

In order to show that int(conv(X)) ≠ ∅, we need to prove that the interior of the convex hull of the given set X is not an empty set. That is, there must exist a point that is interior to the convex hull of X.

Let X = {x_1, x_2, ..., x_{d+1}} be the set of d+1 affinely independent points in R^d. The convex hull of X is defined as the set of all convex combinations of the points in X. Hence, the convex hull of X is given by:

conv(X) = {t_1 x_1 + t_2 x_2 + ... + t_{d+1} x_{d+1} | t_1, t_2, ..., t_{d+1} ≥ 0 and t_1 + t_2 + ... + t_{d+1} = 1}

Now, let us consider the vector v = (1, 1, ..., 1) ∈ R^{d+1}. Note that the sum of the components of v is (d+1), which is equal to the number of points in X. Hence, we can write v as a convex combination of the points in X as follows:

v = (d+1)/∑i=1^{d+1} t_i (x_i)

where t_i = 1/(d+1) for all i ∈ {1, 2, ..., d+1}.

Note that t_i > 0 for all i and t_1 + t_2 + ... + t_{d+1} = 1, which satisfies the definition of a convex combination. Also, we have ∑i=1^{d+1} t_i = 1, which implies that v is in the convex hull of X. Hence, v ∈ conv(X).

Now, let us show that v is an interior point of conv(X). For this, we need to find an ε > 0 such that the ε-ball around v is completely contained in conv(X). Let ε = 1/(d+1). Then, for any point u in the ε-ball around v, we have:

|t_i - 1/(d+1)| ≤ ε for all i ∈ {1, 2, ..., d+1}

Hence, we have t_i ≥ ε > 0 for all i ∈ {1, 2, ..., d+1}. Also, we have:

∑i=1^{d+1} t_i = 1 + (d+1)(-1/(d+1)) = 0

which implies that the point u = ∑i=1^{d+1} t_i x_i is a convex combination of the points in X. Hence, u ∈ conv(X).

Therefore, the ε-ball around v is completely contained in conv(X), which implies that v is an interior point of conv(X). Hence, int(conv(X)) ≠ ∅.

Learn more about independent points here :-

https://brainly.com/question/31987907

#SPJ11

Cos(x), where x is in radians, can be defined by the following infinite series: cos(x)=∑ n=0
[infinity]

(2n)!
(−1) n
x 2n

=1− 2!
x 2

+ 4!
x 4

− 6!
x 6

+ 8!
x 8

+⋯ Carry your answers for parts a,b, and c below to six decimal places. x= 4
π

a) What is the value of cos(π/4) if the series is carried to three terms? b) What is the value of cos(π/4) if the series is carried to four terms? c) What is the approximate absolute error, E A

, for your estimation of cos(π/4) ? d) What is the approximate relative error, ε A

, for your estimation, as a percentage? Carry this answer to 3 significant figures. 3.14 The velocity of a flow may be measured using a manometer, a pitot-static tube, and the following formula: V= rho
2∗γ∗h


where γ is the specific weight of the manometer fluid, h is the differential height in the manometer legs, and rho is the density of the flowing fluid. Given γ=57.0±0.15lb/ft 3
,h=0.15±0.01ft, and rho=0.00238 ±0.0001slug/ft 3
, determine the speed of the flow and its uncertainty. Perform both exact and approximate analyses and present your answers in absolute and relative form.

Answers

The value of cos(π/4) when the series is carried to three terms is 0.707107, the value of cos(π/4) when the series is carried to four terms is 0.707103 and the approximate relative error for the estimation of cos(π/4) is 0.000565%.

a) To find the value of cos(π/4) using the series expansion, we can substitute x = π/4 into the series and evaluate it to three terms:

cos(π/4) = 1 - (2!/(π/4)^2) + (4!/(π/4)^4)

Calculating each term:

2! = 2

(π/4)^2 = (3.14159/4)^2 = 0.61685

4! = 24

(π/4)^4 = (3.14159/4)^4 = 0.09663

Now, plugging the values into the series:

cos(π/4) ≈ 1 - 2(0.61685) + 24(0.09663) = 0.707107

Therefore, the value of cos(π/4) when the series is carried to three terms is approximately 0.707107.

b) To find the value of cos(π/4) using the series expansion carried to four terms, we include one more term in the calculation:

cos(π/4) ≈ 1 - 2(0.61685) + 24(0.09663) - ...

Calculating the next term:

6! = 720

(π/4)^6 = (3.14159/4)^6 = 0.01519

Now, plugging the values into the series:

cos(π/4) ≈ 1 - 2(0.61685) + 24(0.09663) - 720(0.01519) = 0.707103

Therefore, the value of cos(π/4) when the series is carried to four terms is approximately 0.707103.

c) The approximate absolute error, EA, for the estimation of cos(π/4) can be calculated by comparing the result obtained in part b with the actual value of cos(π/4), which is √2/2 ≈ 0.707107.

EA = |0.707107 - 0.707103| ≈ 0.000004

Therefore, the approximate absolute error for the estimation of cos(π/4) is approximately 0.000004.

d) The approximate relative error, εA, for the estimation can be calculated by dividing the absolute error (EA) by the actual value of cos(π/4) and multiplying by 100 to express it as a percentage.

εA = (EA / 0.707107) * 100 ≈ (0.000004 / 0.707107) * 100 ≈ 0.000565%

Therefore, the approximate relative error for the estimation of cos(π/4) is approximately 0.000565%.

To know more about relative error, visit:

https://brainly.com/question/30403282#

#SPJ11

Other Questions
Algoma Incorporated has a capital structure which is based on 25 % debt, 15 % preferred stock, and 60 % common stock. The after-tax cost of debt is 8 %, the cost of preferred is 9 %, and the cost of common stock is 10%. The company is considering a project that is equally as risky as the overall firm. This project has initial costs of $140,000 and cash inflows of $90,000 a year for two years. What is the projected net present value of this project?$17,146.07$18,427.44$19,074.82$21,332.98$17,571.58 "How do they cut the wool." asked Jyoti. what are three risky behaviors that contribute to the current sti epidemic? A.Suppose a 12% rise in the price of coke has reduced its quantity sold by 30% demand for Pepsi has gone up by 26%.Compute the cross elasticity of demand between coke and Pepsi.B.Consider the following statement:If wishes were horses fools would rideIf fishes were horses beggars would rideWho, fools or beggars, in this statement represent the preferences of an economically rational individual? Many partitioning clustering algorithms that automatically determine the number of clusters claim that this is an advantage. List 2 situations in which this is not the case.(1) One situation that is not an advantage for automatic clustering is when the number of clusters calculated is greater than the system can handle. (2) The second situation that is not an advantage for automatic clustering when the data set is known and therefore running the algorithm doesnt not return any additional information. recall that hexadecimal numbers are constructed using the 16 digits 0, 1, 2, 3, 4, 5, 6, 7, 8, 9, a, b, c, d, e, f. (a) how many strings of hexadecimal digits consist of from one through three digits? the sum of the squared deviation scores is ss = 20 for a population of n = 5 scores. what is the variance for this population? group of answer choices 4 5 80 100 A). Reformat of income statement according to contribution format C) Going back bo the original data, the team speculates that they might be able to achseve profitability without changing the sales price if they were to reduce the cost of materials used in manufacture. If the direct materials cost were reduced by eighty cents per unit, how many units would have to be sold i) to break even? ii) to cam a profit of $25,000? D) Again with original data, tho tcam speculates that the problem might lie in inadequate promotion. They want to know by how mach they could increase advertising and still allow the company to carn a target profit of 5% of sales in sales of 60,000 units. E) Going back again to the original data, the tean considers the possibility of covering losses andior generating profit through special ofders. The coespany has been approached by an overseas distributor who wants to parchase 10,000 anits en a special price basis. (These overseas sales would have no effect on regular domestic business.) There would be no sales commission on these sales, shipping costs woeld increase by 50%, while vatiable administrative costs would be reduced by 25%. In addition, a 55,000 insurance foe would have to be paid to cover the goods while in transit. What price would Carolina have to quote on the special order in order to realize a profit of $16,000 an total operations? Would you advise the team to parsue this possibility? Why or why not? Toronto Food Services is considering installing a new refrigeration system that will cost $700,000. The system will be depreciated at a rate of 20% (Class 8) per year over the systems five-year life and then it will be sold for $90,000. The new system will save $250,000 per year in pre-tax operating costs. An initial investment of $70,000 will have to be made in working capital. The tax rate is 35% and the discount rate is 10%. Calculate the NPV of the new refrigeration system. show all calculations An object placed 50cm away from an emerging lens of focal length 15cm produce a focus image on a screen calculate the distance between the object and screen Prove that A search always finds the optimal goal. Recall that A uses an admissible heuristic. Show all the steps of the proof and justify every step. Find sin,sec, and cot if tan= 16/63sin=sec=cot= Select the true statement. The four most prevalent elements in biosystems, in random order, are. H.O.S.C. The chiral amino acids found in biosystems are D-stereoisomers. Hydrogen bonds are weaker in non-polar solvents than they are in water. The strength of ionic bonds is inversely proportional to the dielectric constant of the solvent. Review all the application case studies in Chapter 4 Application Case Studies 4.1 through 4.7 (Business Intelligence, Analytics, and Data Science: A Managerial Perspective). Think about additional examples of organizations or situations where data mining could be used? What problems are these companies most likely solve using data mining? What do you think are the main reasons for data minings popularity the bottom 80 percent of the families in the united states receive approximately ______ percent of total income. The rent control agency of New York City has found that aggregate demand is QD = 160 - 8P. Quantity is measured in tens of thousands of apartments. Price, the average monthly rental rate, is measured in hundreds of dollars. The agency also noted that the increase in Q at lower P results from more three-person families coming into the city from Long Island and demanding apartments. The citys board of realtors acknowledges that this is a good demand estimate and has shown that supply is QS = 70 + 7P.If both the agency and the board are right about demand and supply, what is the free-market price? What is the change in city population if the agency sets a maximum average monthly rent of $300 and all those who cannot find an apartment leave the city?Suppose the agency bows to the wishes of the board and sets a rental of $900 per month on all apartments to allow landlords a "fair" rate of return. If 50 percent of any long-run increases in apartment offerings comes from new construction, how many apartments are constructed? Name the dependent and independent variables for eachprocedure?What must be included in the title of a graph?What is a curve in graphs? A student is taking a multi choice exam in which each question has 4 choices the students randomly selects one out of 4 choices with equal probability for each question assuming that the students has no knowledge of the correct answer to any of the questions.A) what is the probability that the students will get all answers wrong0.2370.316.25noneB) what is the probability that the students will get the questions correct?0.0010.0310.316noneC) if the student make at least 4 questions correct, the students passes otherwise the students fails. what is the probability?0.0160.0150.0010.089D) 100 student take this exam with no knowledge of the correct answer what is the probability that none of them pass0.2080.00010.221none Construct a function that expresses the relationship in the following statement. Use k as the constant of variation. The cost of constructing a silo, A, varies jointly as the height, s, and the radius, v. Missing amounts from financial statements The financial statements at the end of Wolverine Realty's first month of operations are as follows: By analyzing the interrelationships among the four financial statements, determine the proper amounts for the missing items. Use the minus sign to indicate cash outflows, cash payments, and decreases in cash in the Statement of Cash Flows. Wolver Balan April Cash Supplies Land Accounts payable Common stock Retained earnings Total stockholders' equity Total liabilities and stockholders' equity Wolverine Realty Statement of Cash Flows For the Month Ended April 30, 20 Yo Cash flows from (used for) operating activities: Cash received from customers Cash paid for expenses and to creditors Net cash flows from operating activities Cash flows from (used for) investing activities: Cash paid for land Cash flows from (used for) financing activities: Cash received from issuing common stock Cash paid for dividends Net cash flows from financing activities Net increase (decrease) in cash Cash balance, April 1, 20 YO Cash balance, April 30, AYO